LSAT and Law School Admissions Forum

Get expert LSAT preparation and law school admissions advice from PowerScore Test Preparation.

 Jeremy Press
PowerScore Staff
  • PowerScore Staff
  • Posts: 1000
  • Joined: Jun 12, 2017
|
#85740
JocelynL wrote: Thu Mar 11, 2021 9:45 pm
Jeremy Press wrote: Tue Mar 09, 2021 1:26 pm Hi Jocelyn,

I'll give it a try, but note that Answer Choice (A) doesn't fall neatly into a diagram because it is not built on conditional or formal logic statements.

Answer Choice (A): This answer doesn't employ conditional reasoning or formal logic. There are two premises: (1) Jorge says that today's pollution levels will lead to many species of migratory birds becoming extinct; (2) it is widely known pollution might not be reduced at all. Neither of these premises is conditional or formal logic driven. The conclusion is also not conditional, telling us only what the probability is that Jorge believes the extinctions will occur

Answer Choice (B):
  • Premise: Soil Poorly Drained :arrow: Raspberry Bushes Grow Well
  • Premise: Bo says Soil Poorly Drained
  • Conclusion: Bo probably believes Raspberry Bushes Grow Well
Answer Choice (C):
  • Premise: People who Say Coercive Force Justified :most: Believe Gov't Should be Abolished
  • Premise: Wanda says Coercive Force Justified
  • Premise: Wanda probably Believe Gov't Should be Abolished
Answer Choice (D):
  • Premise: Good at Math :most: Good at Chess
  • Premise: Chem Professors says Chemists :most: Good at Math
  • Conclusion: Chem Professor might believe Chemists :most: Good at Chess
Answer Choice (E):
  • Premise: Dr. Bowder says Eating Garlic CAUSES Increased Alertness
  • Premise: Dr. Bowder knows Improve Circulation :arrow: Increase Alertness
  • Conclusion: Dr. Bowder probably believes Eating Garlic CAUSES Increased Circulation
This was extremely helpful Jeremy, thank you! I didn't even see causality in answer choice E. But I see it now, its because of the active word "increases", which makes something else happen.
I diagrammed the first premise in answer choice C as Coercive force :arrow: NOT justified. But you just diagrammed it as a regular statement. I'm afraid I'm forcing conditionality in a lot of these statements (something I need to be mindful of). Thanks again!
Hey Jocelyn,

Just a little note that in going through this question with a student recently (and having a little more time to reflect on it), I actually think you can push answer choice E into a conditional reasoning model. But it's distinguished from the stimulus by the fact that the argument in E is projecting a Mistaken Reversal onto Dr. Bowder's thinking (rather than the stimulus, which is projecting an application of the conditional statement "as is"). I made a change to the diagram of answer choice E in my post above that reflects this.

Hope this helps again!
 hope
  • Posts: 84
  • Joined: Feb 13, 2018
|
#91084
Given Jeremy Press' diagrams of all answer choices, C and D seem to be following the pattern identified by James Finch's post. So are C and D incorrect because they both have the word "most" when the Stim and B do not? Thank you. :-?
 Adam Tyson
PowerScore Staff
  • PowerScore Staff
  • Posts: 5153
  • Joined: Apr 14, 2011
|
#91530
Answer C is incorrect because of the "most" in the premise, Hope. The stimulus had that element of probability only in the conclusion, while the premises were absolute statements.

Answer D is incorrect because it introduces the element of "might" in the conclusion as well as "most" in both premises. Nothing about this answer matches the stimulus!

Get the most out of your LSAT Prep Plus subscription.

Analyze and track your performance with our Testing and Analytics Package.